Đến nội dung

Hình ảnh

Topic về bất đẳng thức

* * * * * 16 Bình chọn

  • Please log in to reply
Chủ đề này có 206 trả lời

#21
truclamyentu

truclamyentu

    Sĩ quan

  • Thành viên
  • 333 Bài viết

giúp mình bài này với, dạo này tư duy kém quá, đây là bài thi thử đại học khối A dành cho khối 10 tham dự
Cho a,b,c Hình đã gửi 0; a + b + c Hình đã gửi 3.Chứng minh rằng:
$\dfrac{1}{1+a} + \dfrac{1}{1+b} + \dfrac{1}{1+c} \geq \dfrac{a}{1+ a^{2} } + \dfrac{b}{1+b^{2} } + \dfrac{c}{1+ c^{2} }$


bất đẳng thức cần cm tương đương với:

$\begin{array}{l}\sum {\dfrac{{a - 1}}{{(a + 1)({a^2} + 1)}}} \le 0\\\\\sum {\dfrac{{a - 1}}{{(a + 1)({a^2} + 1)}}} = \dfrac{1}{4}\sum {\dfrac{{4(a - 1)}}{{(a + 1)({a^2} + 1)}}} \\\\= \sum {(a - 1)} + \dfrac{1}{4}\sum {\left( {\dfrac{{4(a - 1)}}{{(a + 1)({a^2} + 1)}} - (a - 1)} \right)} \\\\= \sum {(a - 1)} - \dfrac{1}{4}\sum {\left( {\dfrac{{{{(a - 1)}^2}({a^2} + 2a + 3)}}{{(a + 1)({a^2} + 1)}}} \right)} \\\\ \le \sum {(a - 1)} \le 0\end{array}$

từ đó suy ra DPCM.

Bài viết đã được chỉnh sửa nội dung bởi truclamyentu: 15-05-2012 - 19:11


#22
anhtuanDQH

anhtuanDQH

    Thượng sĩ

  • Thành viên
  • 236 Bài viết

nhận lời mời dự tiệc mà giờ mới vô được
bài 11
cho các số thực dương a, b, c tm a+b+c=1
CmR:
$\sqrt{a+\dfrac{(b-c)^2}{4}}+\sqrt{b}+\sqrt{c} \leq \sqrt{3} $

olympic toán nữ sinh TQ

--------
bài này có vẻ cũ nhưng hay :(

Cauchy Schwarz
$\ VT^2 \leq 3[a+ \dfrac{ (b-c)^2+2( \sqrt{b} + \sqrt{c} )^2}{4} ] =3[a + \dfrac{ (b-c)^2+2(b+c) + 4\sqrt{bc}} {4} $
Mà $\ (b-c)^2+4\sqrt{bc} =(\sqrt{b}-\sqrt{c})^2(\sqrt{b}+\sqrt{c})^2 +4\sqrt{bc} $
$\leq 2(b+c)(\sqrt{b}-\sqrt{c})^2 +4\sqrt {bc} \leq 2(\sqrt{b}-\sqrt{c})^2+4\sqrt {bc} =2(b+c) $
Suy ra $\ VT^2 \leq 3 $ suy ra ÐPCM

Bài viết đã được chỉnh sửa nội dung bởi dark templar: 24-05-2011 - 09:06
Cần gõ cách dòng cho dễ nhìn

Xăng có thể cạn, lốp có thể mòn..xong số máy số khung thì không bao giờ thay đổi

NGUYỄN ANH TUẤN - CHỦ TỊCH HIỆP HỘI
Hình đã gửi


#23
Zaraki

Zaraki

    PQT

  • Phó Quản lý Toán Cao cấp
  • 4273 Bài viết

Đưa tiếp 1 bài lên vậy :(
Bài 10: Cho $a,b,c>0;a+b+c=3$.Chứng minh rằng:
$\dfrac{bc}{\sqrt{a^2+3}}+\dfrac{ac}{\sqrt{b^2+3}}+\dfrac{ab}{\sqrt{c^2+3}} \le \dfrac{3}{2}$

Mình xin đưa lời giải khác cho bài này!

BĐT $\dfrac{2bc}{a+3}+\dfrac{2ac}{b+3}+\dfrac{2ab}{c+3}\leq\dfrac{3}{2}$ chứng mình khá là dễ.
$\sum\dfrac{2bc}{a+3}=\sum\dfrac{2bc}{a+b+a+c}\le\sum\dfrac{2bc}{4}\left(\dfrac{1}{a+b}+\dfrac{1}{a+c}\right) =\sum\dfrac{1}{2}\left(\dfrac{bc}{a+b}+\dfrac{bc}{a+c}\right) =\dfrac{1}{2}\sum\left(\dfrac{ac}{a+c}+\dfrac{bc}{a+c}\right) =\dfrac{1}{2}\sum a=\dfrac{3}{2}$. (đpcm)

Discovery is a child’s privilege. I mean the small child, the child who is not afraid to be wrong, to look silly, to not be serious, and to act differently from everyone else. He is also not afraid that the things he is interested in are in bad taste or turn out to be different from his expectations, from what they should be, or rather he is not afraid of what they actually are. He ignores the silent and flawless consensus that is part of the air we breathe – the consensus of all the people who are, or are reputed to be, reasonable.

 

Grothendieck, Récoltes et Semailles (“Crops and Seeds”). 


#24
dark templar

dark templar

    Kael-Invoker

  • Hiệp sỹ
  • 3788 Bài viết

Mình xin đưa lời giải khác cho bài này!

BĐT $\dfrac{2bc}{a+3}+\dfrac{2ac}{b+3}+\dfrac{2ab}{c+3}\leq\dfrac{3}{2}$ chứng mình khá là dễ.
$\sum\dfrac{2bc}{a+3}=\sum\dfrac{2bc}{a+b+a+c}\le\sum\dfrac{2bc}{4}\left(\dfrac{1}{a+b}+\dfrac{1}{a+c}\right) =\sum\dfrac{1}{2}\left(\dfrac{bc}{a+b}+\dfrac{bc}{a+c}\right) =\dfrac{1}{2}\sum\left(\dfrac{ac}{a+c}+\dfrac{bc}{a+c}\right) =\dfrac{1}{2}\sum a=\dfrac{3}{2}$. (đpcm)

Chứng mình BĐT này thì có liên quan gì tới Bài 10 đâu :(
P/s:Mong các bạn hãy đọc rõ quy định của topic là giải xong mới được post tiếp nên mong các bạn đừng post thêm bài nữa,trừ khi đã giải được các bài cũ hơn.Thân.

Bài viết đã được chỉnh sửa nội dung bởi dark templar: 24-05-2011 - 09:04

"Do you still... believe in me ?" Sarah Kerrigan asked Jim Raynor - Starcraft II:Heart Of The Swarm.

#25
alex_hoang

alex_hoang

    Thượng úy

  • Hiệp sỹ
  • 1152 Bài viết

Bài 6: Cho $a,b,c \ge 0;ab+bc+ca=3$.Chứng minh rằng:$\dfrac{9}{10} \le \dfrac{1}{a^2+2}+\dfrac{1}{b^2+2}+\dfrac{1}{c^2+2} \le 1$
(Thái Nguyễn Hưng)

bai 6 minh giai nhu sau
bat dang thuc ben phai truoc
ta co $\dfrac{1}{2+ a^{2} } + \dfrac{1}{2+ b^{2} } + \dfrac{1}{2+ c^{2} } \leq 1 \Leftrightarrow \dfrac{ a^{2}}{2+ a^{2} } + \dfrac{ b^{2}}{2+b^{2} } + \dfrac{ c^{2} }{2+c^{2} } \geq 1$
ap dung bdt cauchy schwarz ta co $VT \geq \dfrac{ (a+b+c)^{2}}{6+ a^{2}+ b^{2}+c^{2} } $
su dung dieu kien $ab+bc+ca=3$ ta co dpcm
bdt ben trai
qui dong va rut gon ta duoc bdt tuong duong
$4( a^{2} +b^{2}+ c^{2} )+48 \geq 9a^2b^2c^2+8\sum a^2b^2 $
$\Leftrightarrow 4(a^{2} +b^{2}+ c^{2} )+16abc(a+b+c) \geq 9a^2b^2c^2+24$ (do$ab+bc+ca=3$)
su dung bdt schur la
$a^{2} +b^{2}+ c^{2}+ \dfrac{9abc}{a+b+c} \geq 2( ab+bc+ca)$
ta duoc
$16abc(a+b+c) \geq 9 (abc)^{2}+\dfrac{36abc}{a+b+c}$
de thay bdt tren dung (ngai viet qua nen con lai tu cm)

Bài viết đã được chỉnh sửa nội dung bởi wallunint: 25-05-2011 - 08:25

alex_hoang


HẸN NGÀY TRỞ LẠI VMF THÂN MẾN

http://www.scribd.co...oi-Ban-Cung-The

#26
Zaraki

Zaraki

    PQT

  • Phó Quản lý Toán Cao cấp
  • 4273 Bài viết
Cho mình hỏi luôn một bài tương tự bài 10
Bài 14: Cho các số thực dương $a,b,c$ sao cho $abc=1$.CMR:
$\dfrac{a}{\sqrt{b^{2}+3}}+\dfrac{b}{\sqrt{c^{2}+3}}+\dfrac{c}{\sqrt{a^{2}+3}}\ge \dfrac{3}{2}$

Bài viết đã được chỉnh sửa nội dung bởi dark templar: 28-05-2011 - 17:28
Gõ Latex hoàn toàn trong bài viết

Discovery is a child’s privilege. I mean the small child, the child who is not afraid to be wrong, to look silly, to not be serious, and to act differently from everyone else. He is also not afraid that the things he is interested in are in bad taste or turn out to be different from his expectations, from what they should be, or rather he is not afraid of what they actually are. He ignores the silent and flawless consensus that is part of the air we breathe – the consensus of all the people who are, or are reputed to be, reasonable.

 

Grothendieck, Récoltes et Semailles (“Crops and Seeds”). 


#27
alex_hoang

alex_hoang

    Thượng úy

  • Hiệp sỹ
  • 1152 Bài viết

Tiếp tục nào :(
Bài 9: Cho các số thực không âm $a,b,c$ thỏa mãn$a+b+c=2$. CMR:

$\dfrac{{bc}}{{{a^2} + 1}} + \dfrac{{ac}}{{{b^2} + 1}} + \dfrac{{ab}}{{{c^2} + 1}} \leqslant 1$


*Nhận xét : Bài này có khá nhiều cách giải rất trâu bò
Các bạn có thể tìm đc bao nhiêu cách giải cho bài này ??? :(
Các bạn nhanh chóng giải bài này đi ;))
Gợi ý : Chỉ sử dụng bất đẳng thức $AM-GM$ $(Cauchy)$

Đừng post bài nữa nhá :D để giải xong mấy bài trên rồi post tiếp nha :D


mình giải bài 9 này như sau :
$\dfrac{bc}{1+ a^{2} } \leq \dfrac{bc(b+c)}{ab(a+b)+bc(b+c)+ca(c+a)}$
Thật vậy:
$\Leftrightarrow (b+c)(1+ a^{2} \geq bc(b+c)+a(b^{2}+ c^{2})+ a^{2}(b+c)$
vì $ a(b^{2}+ c^{2})=a(b+c)^{2}-2abc$
$ \Rightarrow (b+c)(1+a^{2})-a( b+c)^{2})- a^{2}(b+c) \geq bc(b+c)-2abc$
$ \Leftrightarrow (b+c)(1-ab-ac) :( bc(b+c-2a)$
Nếu $1-ab-ac \geq 0$ thì hiển nhiên đúng nếu $ b+c \leq 2a$
Ngược lại
$VT-VP \geq (b+c)(1-ab-ac)-(b+c-2a) \dfrac{(b+c)^{2} }{4}= \dfrac{bc( a)^{2} }{4} \geq 0$
Vậy bất đẳng thức đã được chứng minh. :D

Bài viết đã được chỉnh sửa nội dung bởi wallunint: 25-05-2011 - 08:45
Gõ Latex hoàn toàn trong bài viết

alex_hoang


HẸN NGÀY TRỞ LẠI VMF THÂN MẾN

http://www.scribd.co...oi-Ban-Cung-The

#28
alex_hoang

alex_hoang

    Thượng úy

  • Hiệp sỹ
  • 1152 Bài viết

Cho mình hỏi luôn một bài tương tự bài 10
Bài 14: Cho các số thực dương $a,b,c$ sao cho $abc=1$.CMR:
$\dfrac{a}{\sqrt{b^{2}+3}}+\dfrac{b}{\sqrt{c^{2}+3}}+\dfrac{c}{\sqrt{a^{2}+3}}\ge \dfrac{3}{2}$


bai 14 ban vua neu minh cm nhu sau
do $abc=1 \Rightarrow ab+bc+ca \geq 3$
vay ta se cm bdt manh hon nhu sau
$\dfrac{2a}{ \sqrt{(b+c)(b+a} }+ \dfrac{2b}{ \sqrt{(b+c)(c+a} }+ \dfrac{2c}{ \sqrt{(a+c)(b+a} } \geq 3$
that vay
ta dat $u= \sqrt{b+c},v= \sqrt{a+c},w= \sqrt{a+b}$
khi do bdt tuong duong
$ \dfrac{ v^{2}+ w^{2} - u^{2} }{uw} + \dfrac{u^{2}+ w^{2} - v^{2} }{uv}+\dfrac{ u^{2}+ w^{2} - v^{2} }{vw} \geq 3$
tu day qui dong va thu gon ap dung AM GM ta co dpcm

Bài viết đã được chỉnh sửa nội dung bởi dark templar: 28-05-2011 - 17:28
Gõ Latex hoàn toàn trong bài viết

alex_hoang


HẸN NGÀY TRỞ LẠI VMF THÂN MẾN

http://www.scribd.co...oi-Ban-Cung-The

#29
l.kuzz.l

l.kuzz.l

    Trung sĩ

  • Thành viên
  • 109 Bài viết
Này thì BĐT
Bài 18 (Mãi chưa ai giải quyêt)
Cho các số thực x,y,z t/m xyz=1
CMR với mọi số thực m ta có $ \dfrac{(x+m)^2 }{(x-1)^2}+ \dfrac{(y+m)^2}{(y-1)^2}+ \dfrac{(z+m)^2}{(z-1)^2} \geq 1$
Bài 19 Cho các số thực dương t/m x.(x+y+z)=3yz
CMR $ (x+y)^3 + (x+z)^3 + 3(x+y)(y+z)(x+z) \leq 5(y+z)^3 $
Bài 20 Cho 3 số thực dương x,y,z t/m xyz=1 CMR 2A :D 1, 2B :( 1
Với $ A= \dfrac{1}{2x^2+y^2=3} + \dfrac{1}{2y^2+z^2+3}+ \dfrac{1}{2z^2+x^2+3} $
$ B= \dfrac{1}{(x+1)^2+y^2+1}+ \dfrac{1}{(y+1)^2+z^2+1}+ \dfrac{1}{(z+1)^2+x^2+1}$
Tất nhin là CM A=>B
Bài 21 Cho các sô thực không âm $a_{1}, a_{2}, a_{3},..,:($
CMR $a_{1}.a_{2}...;)) +1\leq \sqrt[n]{ (a_{1}+1)(a_{2}+1)..(:(+1)}$
Bài 22 Cho các số thực không âm x,y,z,m,n,p
CMR $ \sqrt[3]{mnp}+ \sqrt[3]{xyz} \leq \sqrt[3]{(x+m)(y+n)(z+n)}$
P/s Toàn những bài ôn chuyên ấy mỗi tội chưa có lời giải
Tạm thời như vậy chứ còn nhìu lắm
Anh nào vào sửa hộ em nha

Bài viết đã được chỉnh sửa nội dung bởi khacduongpro_165: 25-05-2011 - 12:18

Chúng ta không thể biết chính xác 100% việc sẽ xảy ra trong tương lai
Và đây là điều duy nhất ta có thể biết 100% trong tương lai


#30
truclamyentu

truclamyentu

    Sĩ quan

  • Thành viên
  • 333 Bài viết

Bài 20 Cho 3 số thực dương x,y,z t/m xyz=1 CMR 2A :( 1, 2B ;)) 1
Với $ A= \dfrac{1}{2x^2+y^2=3} + \dfrac{1}{2y^2+z^2+3}+ \dfrac{1}{2z^2+x^2+3} $
$ B= \dfrac{1}{(x+1)^2+y^2+1}+ \dfrac{1}{(y+1)^2+z^2+1}+ \dfrac{1}{(z+1)^2+x^2+1}$


áp dung AM-GM TA có :

${x^2} + {y^2} \ge 2xy;{x^2} + 1 \ge 2x$

tương tự suy ra :

$2A \le \dfrac{1}{{xy + x + 1}} + \dfrac{1}{{yz + y + 1}} + \dfrac{1}{{zx + z + 1}}\\\\= \dfrac{1}{{\dfrac{1}{z} + x + 1}} + \dfrac{1}{{\dfrac{z}{{xz}} + \dfrac{1}{{xz}} + 1}} + \dfrac{1}{{zx + z + 1}}\\\\ = \dfrac{z}{{xz + x + 1}} + \dfrac{{xz}}{{xz + x + 1}} + \dfrac{1}{{zx + z + 1}} = 1$

chứng minh 2B :( 1 hoàn toàn tương tự :D :(

Bài viết đã được chỉnh sửa nội dung bởi truclamyentu: 24-05-2011 - 23:23


#31
alex_hoang

alex_hoang

    Thượng úy

  • Hiệp sỹ
  • 1152 Bài viết
[quote name='alex_hoang' date='May 25 2011, 12:55 AM' post='262026']
bai 18 nay kha hay no la bai tong quat cua bai BDT trong ki thi imo 2008
minh giai bai 18 nhu sau
dat $a= \dfrac{x+m}{x-1},b= \dfrac{y+m}{y-1},c= \dfrac{z+m}{z-1}$
khi do ta co
$x= \dfrac{a+m}{a-1}, y= \dfrac{b+m}{b-1}, z= \dfrac{c+m}{c-1}$,
do $xyz=1$ nen $(a+m)(b+m)(c+m)=(a-1)(b-1)(c-1)$
$ \Leftrightarrow 1=-(ab+bc+ca)+(1-m)(a+b+c)+m- m^{2}$
ta co
$ a^{2}+ b^{2}+ c^{2}=2(-(ab+bc+ca)+(1-m)(a+b+c)+m- m^{2} )-1$
$(a+b+c+m-1) ^{2}+ m^{2} \geq 0$
vay bdt da dc cm

Bài viết đã được chỉnh sửa nội dung bởi wallunint: 25-05-2011 - 12:17
latex

alex_hoang


HẸN NGÀY TRỞ LẠI VMF THÂN MẾN

http://www.scribd.co...oi-Ban-Cung-The

#32
nguyen thai phuc

nguyen thai phuc

    Sĩ quan

  • Thành viên
  • 430 Bài viết

Dạo này nghỉ hè rỗi rãi sáng tác 1 bài tặng các bạn nhân dịp cuối năm học :D
Cho $a,b,c>0 $ thỏa mãn $ a^2+b^2+c^2=1 $.Chứng minh:
$ \dfrac{c+ab}{\sqrt{1-a^2}}+\dfrac{a+bc}{\sqrt{1-b^2}}+\dfrac{b+ca}{\sqrt{1-c^2}} \le \sqrt{6+3a+3b+3c} $

Lâu lắm mới vào vmf, làm tí cho xôm tụ :D.
Ta có:
$\begin{align} & \sqrt{\dfrac{{{\left( c+ab \right)}^{2}}}{1-{{a}^{2}}}}=\sqrt{\dfrac{{{c}^{2}}+{{a}^{2}}{{b}^{2}}+2abc}{{{b}^{2}}+{{c}^{2}}}}=\sqrt{\dfrac{\left( {{c}^{2}}+{{b}^{2}} \right)\left( {{c}^{2}}+{{a}^{2}} \right)+2abc}{{{b}^{2}}+{{c}^{2}}}} \\ & \le \sqrt{{{c}^{2}}+{{a}^{2}}+a} \\ \end{align}$
vậy $\sum{\sqrt{\dfrac{{{\left( c+ab \right)}^{2}}}{1-{{a}^{2}}}}}\le \sum{\sqrt{{{c}^{2}}+{{a}^{2}}+a}}\le \sqrt{6+3a+3b+3c}$
P/S: mấy cái (1);(2);(3) ở đâu ra ấy nhỉ ^_^

Bài viết đã được chỉnh sửa nội dung bởi dark templar: 28-05-2011 - 17:26

Hình đã gửi

#33
NguyThang khtn

NguyThang khtn

    Thượng úy

  • Hiệp sỹ
  • 1468 Bài viết
Thư giãn với bài sau:
cho a,b,c là độ dài ba cạnh một tam giác .CMR:
$(a+b+c)abc \geq (ab+bc+ac)\sqrt[3]{\sum(a+b-c)}$

It is difficult to say what is impossible, for the dream of yesterday is the hope of today and the reality of tomorrow

 


#34
NguyThang khtn

NguyThang khtn

    Thượng úy

  • Hiệp sỹ
  • 1468 Bài viết

Thư giãn với bài sau:
cho a,b,c là độ dài ba cạnh một tam giác .CMR:
$(a+b+c)abc \geq (ab+bc+ac)\sqrt[3]{\sum(a+b-c)}$

Lâu không thấy ai giải,mình post lên cho các bạn tham khảo nha !
Đặt $b+c-a=x;a+c-b=y;a+b-c=z$ thì $x,y,z > 0$ và $a=\dfrac{z+y}{2};b=\dfrac{x+z}{2};c=\dfrac{x+y}{2}$
Chuẩn hóa $ xyz=1$.BDT trở thành :
$x+y+z \geq \sum\dfrac{2}{x+y}$
theo AM-GM:
$ \sum\dfrac{2}{x+y} \leq \sum \dfrac{2}{2\sqrt{xy}} = \sum \sqrt{x} \leq \dfrac{x+1+y+1+z+1}{2} \leq x+y+z$
Dấu = xảy ra khi $a=b=c$

Bài viết đã được chỉnh sửa nội dung bởi dark templar: 03-06-2011 - 16:29
Latex+Gõ tiếng Việt có dấu

It is difficult to say what is impossible, for the dream of yesterday is the hope of today and the reality of tomorrow

 


#35
Tạ Hồng Quảng

Tạ Hồng Quảng

    Binh nhì

  • Thành viên
  • 16 Bài viết
Bài 22. Cho a, b, c, d, e không âm. Chứng minh
$a^6b+b^6c+c^6d+d^6e+e^6a\ge abcde(a^2+b^2+c^2+d^2+e^2)$

(Sáng tác)

Bài viết đã được chỉnh sửa nội dung bởi Tạ Hồng Quảng: 13-09-2011 - 19:52


#36
NguyThang khtn

NguyThang khtn

    Thượng úy

  • Hiệp sỹ
  • 1468 Bài viết

Bài 22. Cho a, b, c, d, e không âm. Chứng minh
$a^6b+b^6c+c^6d+d^6e+e^6a\ge abcde(a^2+b^2+c^2+d^2+e^2)$

Theo AM-GM:

$2a^6b+ abcde(+c^2+d^2+e^2) \geq 5a^2.abcde$
Làm tương tự rùi cộng lại ta được ĐPCM!

Bài viết đã được chỉnh sửa nội dung bởi bboy114crew: 05-07-2011 - 11:29

It is difficult to say what is impossible, for the dream of yesterday is the hope of today and the reality of tomorrow

 


#37
NguyThang khtn

NguyThang khtn

    Thượng úy

  • Hiệp sỹ
  • 1468 Bài viết

Các bạn xem lời giải của mình ở đây :Rightarrow

Solution

sao lại ra thế này nhỉ?
$\sqrt{\dfrac{a^3}{a^3+(b+c)^3}}= \sqrt{\dfrac{1}{1+(\dfrac{b+c}{a})^3}} = \sqrt{\dfrac{1}{(1+\dfrac{b+c}{a})(\dfrac{(b+c)^2}{a^2}-\dfrac{b+c}{a}+1)}} \geq \dfrac{1}{\dfrac{(b+c)^2}{a^2}+2} \geq\dfrac{a^2}{2(a^2+b^2+c^2)}$

Bài viết đã được chỉnh sửa nội dung bởi bboy114crew: 19-07-2011 - 18:06

It is difficult to say what is impossible, for the dream of yesterday is the hope of today and the reality of tomorrow

 


#38
Zaraki

Zaraki

    PQT

  • Phó Quản lý Toán Cao cấp
  • 4273 Bài viết
Lâu thấy topic no có bài mới, post một bài lên nè!
$\fbox{23}$. Cho các số không âm $a,b,c$ và $d$ sao cho $a+b+c+d\neq0$. Chứng minh rằng:

$a^{3}+b^{3}+c^{3}+d^{3}+\dfrac{32abcd}{a+b+c+d}\geq3(abc+abd+acd+bcd)$


Discovery is a child’s privilege. I mean the small child, the child who is not afraid to be wrong, to look silly, to not be serious, and to act differently from everyone else. He is also not afraid that the things he is interested in are in bad taste or turn out to be different from his expectations, from what they should be, or rather he is not afraid of what they actually are. He ignores the silent and flawless consensus that is part of the air we breathe – the consensus of all the people who are, or are reputed to be, reasonable.

 

Grothendieck, Récoltes et Semailles (“Crops and Seeds”). 


#39
lethanhtam_dt

lethanhtam_dt

    Binh nhất

  • Thành viên
  • 20 Bài viết
Cho a,b,c là các số dương thỏa mãn $ \sqrt{ab} +\sqrt{bc} +\sqrt{ca} \geq a+b+c$
CMR: $\sqrt{a^2+ab+b^2} +\sqrt{b^2+bc+c^2} +\sqrt{c^2+ca+a^2} \geq \sqrt{3}(a+b+c)$

#40
NguyThang khtn

NguyThang khtn

    Thượng úy

  • Hiệp sỹ
  • 1468 Bài viết
Bài này không bít cái ĐK này:

$\sqrt{ab} +\sqrt{bc} +\sqrt{ca} \geq a+b+c$

để làm gì nhỉ?
Áp dụng BDT:
$\sqrt{x^2+xy+y^2} \geq \dfrac{\sqrt{3}(x+y)}{2}#$
Từ đó ta có ngay:
$\sqrt{a^2+ab+b^2} +\sqrt{b^2+bc+c^2} +\sqrt{c^2+ca+a^2} \geq \sqrt{3}(a+b+c)$

It is difficult to say what is impossible, for the dream of yesterday is the hope of today and the reality of tomorrow

 





1 người đang xem chủ đề

0 thành viên, 1 khách, 0 thành viên ẩn danh